If $a+b+c=abc$ then $sumlimits_{cyc}frac{1}{7a+b}leqfrac{sqrt3}{8}$












9












$begingroup$


Let $a$, $b$ and $c$ be positive numbers such that $a+b+c=abc$. Prove that:
$$frac{1}{7a+b}+frac{1}{7b+c}+frac{1}{7c+a}leqfrac{sqrt3}{8}$$



I tried C-S:
$$left(sum_{cyc}frac{1}{7a+b}right)^2leqsum_{cyc}frac{1}{(ka+mb+c)(7a+b)^2}sum_{cyc}(ka+mb+c)=$$
$$=sum_{cyc}frac{(k+m+1)(a+b+c)}{(ka+mb+c)(7a+b)^2}.$$
Thus, it remains to prove that
$$sum_{cyc}frac{k+m+1}{(ka+mb+c)(7a+b)^2}leqfrac{3}{64abc},$$
but I did not find non-negative values of $k$ and $m$, for which the last inequality is true.



Thank you!










share|cite|improve this question









$endgroup$








  • 4




    $begingroup$
    FOr those fans who hate abbreviations, C-S is Cauchy-Schwartz
    $endgroup$
    – Mark Fischler
    Mar 21 '17 at 20:10






  • 4




    $begingroup$
    As a good start, take $a=sqrt{3}x, b = sqrt{3}y, c = sqrt{3}z$ so that you are working with $x+y+z = 3xyz$ and the extremum will occur at $(1,1,1)$. Makes the horrendous algebra a little cleaner.
    $endgroup$
    – Mark Fischler
    Mar 21 '17 at 22:57


















9












$begingroup$


Let $a$, $b$ and $c$ be positive numbers such that $a+b+c=abc$. Prove that:
$$frac{1}{7a+b}+frac{1}{7b+c}+frac{1}{7c+a}leqfrac{sqrt3}{8}$$



I tried C-S:
$$left(sum_{cyc}frac{1}{7a+b}right)^2leqsum_{cyc}frac{1}{(ka+mb+c)(7a+b)^2}sum_{cyc}(ka+mb+c)=$$
$$=sum_{cyc}frac{(k+m+1)(a+b+c)}{(ka+mb+c)(7a+b)^2}.$$
Thus, it remains to prove that
$$sum_{cyc}frac{k+m+1}{(ka+mb+c)(7a+b)^2}leqfrac{3}{64abc},$$
but I did not find non-negative values of $k$ and $m$, for which the last inequality is true.



Thank you!










share|cite|improve this question









$endgroup$








  • 4




    $begingroup$
    FOr those fans who hate abbreviations, C-S is Cauchy-Schwartz
    $endgroup$
    – Mark Fischler
    Mar 21 '17 at 20:10






  • 4




    $begingroup$
    As a good start, take $a=sqrt{3}x, b = sqrt{3}y, c = sqrt{3}z$ so that you are working with $x+y+z = 3xyz$ and the extremum will occur at $(1,1,1)$. Makes the horrendous algebra a little cleaner.
    $endgroup$
    – Mark Fischler
    Mar 21 '17 at 22:57
















9












9








9


7



$begingroup$


Let $a$, $b$ and $c$ be positive numbers such that $a+b+c=abc$. Prove that:
$$frac{1}{7a+b}+frac{1}{7b+c}+frac{1}{7c+a}leqfrac{sqrt3}{8}$$



I tried C-S:
$$left(sum_{cyc}frac{1}{7a+b}right)^2leqsum_{cyc}frac{1}{(ka+mb+c)(7a+b)^2}sum_{cyc}(ka+mb+c)=$$
$$=sum_{cyc}frac{(k+m+1)(a+b+c)}{(ka+mb+c)(7a+b)^2}.$$
Thus, it remains to prove that
$$sum_{cyc}frac{k+m+1}{(ka+mb+c)(7a+b)^2}leqfrac{3}{64abc},$$
but I did not find non-negative values of $k$ and $m$, for which the last inequality is true.



Thank you!










share|cite|improve this question









$endgroup$




Let $a$, $b$ and $c$ be positive numbers such that $a+b+c=abc$. Prove that:
$$frac{1}{7a+b}+frac{1}{7b+c}+frac{1}{7c+a}leqfrac{sqrt3}{8}$$



I tried C-S:
$$left(sum_{cyc}frac{1}{7a+b}right)^2leqsum_{cyc}frac{1}{(ka+mb+c)(7a+b)^2}sum_{cyc}(ka+mb+c)=$$
$$=sum_{cyc}frac{(k+m+1)(a+b+c)}{(ka+mb+c)(7a+b)^2}.$$
Thus, it remains to prove that
$$sum_{cyc}frac{k+m+1}{(ka+mb+c)(7a+b)^2}leqfrac{3}{64abc},$$
but I did not find non-negative values of $k$ and $m$, for which the last inequality is true.



Thank you!







inequality contest-math






share|cite|improve this question













share|cite|improve this question











share|cite|improve this question




share|cite|improve this question










asked Mar 21 '17 at 20:05









Michael RozenbergMichael Rozenberg

106k1893198




106k1893198








  • 4




    $begingroup$
    FOr those fans who hate abbreviations, C-S is Cauchy-Schwartz
    $endgroup$
    – Mark Fischler
    Mar 21 '17 at 20:10






  • 4




    $begingroup$
    As a good start, take $a=sqrt{3}x, b = sqrt{3}y, c = sqrt{3}z$ so that you are working with $x+y+z = 3xyz$ and the extremum will occur at $(1,1,1)$. Makes the horrendous algebra a little cleaner.
    $endgroup$
    – Mark Fischler
    Mar 21 '17 at 22:57
















  • 4




    $begingroup$
    FOr those fans who hate abbreviations, C-S is Cauchy-Schwartz
    $endgroup$
    – Mark Fischler
    Mar 21 '17 at 20:10






  • 4




    $begingroup$
    As a good start, take $a=sqrt{3}x, b = sqrt{3}y, c = sqrt{3}z$ so that you are working with $x+y+z = 3xyz$ and the extremum will occur at $(1,1,1)$. Makes the horrendous algebra a little cleaner.
    $endgroup$
    – Mark Fischler
    Mar 21 '17 at 22:57










4




4




$begingroup$
FOr those fans who hate abbreviations, C-S is Cauchy-Schwartz
$endgroup$
– Mark Fischler
Mar 21 '17 at 20:10




$begingroup$
FOr those fans who hate abbreviations, C-S is Cauchy-Schwartz
$endgroup$
– Mark Fischler
Mar 21 '17 at 20:10




4




4




$begingroup$
As a good start, take $a=sqrt{3}x, b = sqrt{3}y, c = sqrt{3}z$ so that you are working with $x+y+z = 3xyz$ and the extremum will occur at $(1,1,1)$. Makes the horrendous algebra a little cleaner.
$endgroup$
– Mark Fischler
Mar 21 '17 at 22:57






$begingroup$
As a good start, take $a=sqrt{3}x, b = sqrt{3}y, c = sqrt{3}z$ so that you are working with $x+y+z = 3xyz$ and the extremum will occur at $(1,1,1)$. Makes the horrendous algebra a little cleaner.
$endgroup$
– Mark Fischler
Mar 21 '17 at 22:57












0






active

oldest

votes











Your Answer





StackExchange.ifUsing("editor", function () {
return StackExchange.using("mathjaxEditing", function () {
StackExchange.MarkdownEditor.creationCallbacks.add(function (editor, postfix) {
StackExchange.mathjaxEditing.prepareWmdForMathJax(editor, postfix, [["$", "$"], ["\\(","\\)"]]);
});
});
}, "mathjax-editing");

StackExchange.ready(function() {
var channelOptions = {
tags: "".split(" "),
id: "69"
};
initTagRenderer("".split(" "), "".split(" "), channelOptions);

StackExchange.using("externalEditor", function() {
// Have to fire editor after snippets, if snippets enabled
if (StackExchange.settings.snippets.snippetsEnabled) {
StackExchange.using("snippets", function() {
createEditor();
});
}
else {
createEditor();
}
});

function createEditor() {
StackExchange.prepareEditor({
heartbeatType: 'answer',
autoActivateHeartbeat: false,
convertImagesToLinks: true,
noModals: true,
showLowRepImageUploadWarning: true,
reputationToPostImages: 10,
bindNavPrevention: true,
postfix: "",
imageUploader: {
brandingHtml: "Powered by u003ca class="icon-imgur-white" href="https://imgur.com/"u003eu003c/au003e",
contentPolicyHtml: "User contributions licensed under u003ca href="https://creativecommons.org/licenses/by-sa/3.0/"u003ecc by-sa 3.0 with attribution requiredu003c/au003e u003ca href="https://stackoverflow.com/legal/content-policy"u003e(content policy)u003c/au003e",
allowUrls: true
},
noCode: true, onDemand: true,
discardSelector: ".discard-answer"
,immediatelyShowMarkdownHelp:true
});


}
});














draft saved

draft discarded


















StackExchange.ready(
function () {
StackExchange.openid.initPostLogin('.new-post-login', 'https%3a%2f%2fmath.stackexchange.com%2fquestions%2f2197138%2fif-abc-abc-then-sum-limits-cyc-frac17ab-leq-frac-sqrt38%23new-answer', 'question_page');
}
);

Post as a guest















Required, but never shown

























0






active

oldest

votes








0






active

oldest

votes









active

oldest

votes






active

oldest

votes
















draft saved

draft discarded




















































Thanks for contributing an answer to Mathematics Stack Exchange!


  • Please be sure to answer the question. Provide details and share your research!

But avoid



  • Asking for help, clarification, or responding to other answers.

  • Making statements based on opinion; back them up with references or personal experience.


Use MathJax to format equations. MathJax reference.


To learn more, see our tips on writing great answers.




draft saved


draft discarded














StackExchange.ready(
function () {
StackExchange.openid.initPostLogin('.new-post-login', 'https%3a%2f%2fmath.stackexchange.com%2fquestions%2f2197138%2fif-abc-abc-then-sum-limits-cyc-frac17ab-leq-frac-sqrt38%23new-answer', 'question_page');
}
);

Post as a guest















Required, but never shown





















































Required, but never shown














Required, but never shown












Required, but never shown







Required, but never shown

































Required, but never shown














Required, but never shown












Required, but never shown







Required, but never shown







Popular posts from this blog

Quarter-circle Tiles

build a pushdown automaton that recognizes the reverse language of a given pushdown automaton?

Mont Emei